Азиатско-Тихоокеанская математическая олимпиада, 1989 год


Пусть $x_1$, $x_2$, $\dots$, $x_n$ — положительные числа, $S = x_1 + x_2 + \dots + x_n$. Докажите, что $$ \left( {1 + {x_1}} \right)\left( {1 + {x_2}} \right) \dots \left( {1 + {x_n}} \right) \leq 1 + S + \frac{{{S^2}}}{{2!}} + \frac{{{S^3}}}{{3!}} + \dots + \frac{{{S^n}}}{{n!}}. $$
посмотреть в олимпиаде

Комментарий/решение:

  3
2016-05-07 01:57:59.0 #

$$x_1,x_2,x_3,..,x_n>0$$ $$S=x_1+x_2+x_3+..+x_n$$

$$(1+x_1)(1+x_2)...(1+x_n)\leq 1+S+\frac{S^2}{2!}+\frac{S^3}{3!}+...+\frac{S^n}{n!}$$

$$(1+x_1)(1+x_2)...(1+x_n)\leq\sum \limits_{i=0}^{n}{\frac{S^i}{i!}}$$

По методу математической индукции:

$$1) n=1 \Rightarrow 1+x_1\leq S \Rightarrow x_2+x_3+...+x_n\geq 1$$

$$2) n=k \Rightarrow (1+x_1)(1+x_2)...(1+x_k)\leq\sum \limits_{i=0}^{k}{\frac{S^i}{i!}}$$

$$3) n=k+1 \Rightarrow (1+x_1)(1+x_2)...(1+x_k)(1+x_{k+1})\leq\sum \limits_{i=0}^{k+1}{\frac{S^i}{i!}}\Rightarrow $$

$$\Rightarrow (1+x_1)(1+x_2)...(1+x_k)(1+x_{k+1})\leq(1+x_{k+1})*\sum \limits_{i=0}^{k}{\frac{S^i}{i!}}\leq \sum \limits_{i=0}^{k}{\frac{S^i}{i!}} +\frac{S^{k+1}}{(k+1)!}\Rightarrow$$

$$\Rightarrow(1+x_{k+1})*\sum \limits_{i=0}^{k}{\frac{S^i}{i!}}\leq \sum \limits_{i=0}^{k}{\frac{S^i}{i!}} +\frac{S^{k+1}}{(k+1)!}\Rightarrow (1+x_{k+1})*\sum \limits_{i=0}^{k}{\frac{S^i}{i!}}- \sum \limits_{i=0}^{k}{\frac{S^i}{i!}} \leq \frac{S^{k+1}}{(k+1)!} \Rightarrow$$

$$\Rightarrow x_{k+1}*\sum \limits_{i=0}^{k}{\frac{S^i}{i!}} \leq \frac{S^{k+1}}{(k+1)!} \Rightarrow [B] \Rightarrow x_{k+1}+Sx_{k+1}+\frac{S^2}{2!}x_{k+1}+\frac{S^3}{3!}x_{k+1}+...+\frac{S^k}{k!}x_{k+1} \leq \frac{S^{k+1}}{(k+1)!}$$

Отсюда мы аналогично видим что неравенство $[B]$ справедливо для всех $k\in N$

пред. Правка 2   3
2016-05-07 13:19:54.0 #

Метод №2. И можно доказать данное неравенству в виде $P(x_1,x_2,..,x_n) - R\leq \sum \limits_{i=1}^{n}{\frac{S^i}{i!}}$

$P(x_1,x_2,..,x_n)$ -многочлен от нескольких переменных

$\sum \limits_{i=1}^{n}{\frac{S^i}{i!}}$ - Формула Тейлора

$R$ -остаток в формуле Тейлора (в форме Лагранжа,Пеано,КОШИ, в интегральной форме и т. д.)

  1
2017-03-14 13:51:10.0 #

$$\sqrt[n]{(1+x_1)(1+x_2)...(1+x_n)}\leq \frac{1+x_1+1+x_2+...+1+x_n}{n}=$$ $$=\frac{\underbrace{x_1+x_2+...x_n}_S+n}{n}=\frac{S}{n}+1\Rightarrow$$

$$\Rightarrow(1+x_1)(1+x_2)...(1+x_n)\leq \left(\frac{S}{n}+1\right)^n=$$

$$=1+\sum \limits_{i=1}^{n}{{C_n}^i \left(\frac{S}{n}\right)^i}\leq 1+\sum \limits_{i=1}^{n}{\left(\frac{S}{n}\right)^i}$$